Diễn Đàn MathScope

Diễn Đàn MathScope (http://forum.mathscope.org/index.php)
-   Các Bài Toán Đã Được Giải (http://forum.mathscope.org/forumdisplay.php?f=107)
-   -   Topic về Bất đẳng thức Olympic (http://forum.mathscope.org/showthread.php?t=9330)

Red Devils 01-09-2009 05:20 PM

Topic về Bất đẳng thức Olympic
 
Mình không giỏi BĐT lắm nhưng cũng bạo gan lấp ra Topic này. Topic sẽ tập hợp những bất đẳng thức trong các kì thi Olympiad ở các nước. Có 1 vài quy đinh để Topic thêm đẹp:
1. Ghi rõ nguồn: bài trong kì thi nào, năm bao nhiêu
2. Khi post bài các bạn hãy để ý số bài và ghi rõ.
3. Bạn có thể:
i. Post đề bài và để mọi người giải
ii. Post đề bài + lời giải (của mình hoặc sưu tầm)
4. Các BĐT đều phải là BĐT đã xuất hiện (chính thức hoặc dự tuyển) trong 1 kì thi toán Olympiad nào đó. Ngoài ra các bất đẳng thức xuất hiện trên các tạp chí toán học cũng được chấp nhận.
5. Khi giải bạn hãy trích dẫn lại bài sẽ giải để mọi người tiện theo dõi.
6. Bạn có thể post mở rộng của BĐT nếu muốn.
Bây giờ chúng ta cùng bắt đầu:

Bài toán 1: (IMO Shortlist 2001)
Cho $n $ số thực tuỳ ý $x_{1},x_{2},\ldots,x_{n} $. Chứng minh rằng:
$\frac{x_{1}}{1+x_{1}^{2}}+\frac{x_{2}}{1+x_{1}^{2} +x_{2}^{2}}+\cdots+\frac{x_{n}}{1+x_{1}^{2}+\cdots +x_{n}^{2}}<\sqrt{n} $
Bài toán 2: (IMO Shortlist 2001)
Chứng minh rắng với mọi số thực dương $a, b, c $ ta có:
$\frac{a}{\sqrt{a^{2}+8bc}}+\frac{b}{\sqrt{b^{2}+8c a}}+\frac{c}{\sqrt{c^{2}+8ab}}\geq 1 $
Bài toán 3: (IMO LongList 1967)
Chứng minh rắng với mọi số thực dương $a, b, c $ ta có:
$\frac{1}{a}+\frac{1}{b}+\frac{1}{c}\leq\frac{a^{8} +b^{8}+c^{8}}{a^{3}b^{3}c^{3}} $
Bài toán 4: (14th Turkish Mathematical Olympiad, 2006)
Với mọi số thực dương $a, b, c $ thỏa mãn đẳng thức $a+b + c=1 $. Chứng minh rằng:
$\frac{1}{ab+2c^2+2c}+\frac{1}{bc+2a^2+2a}+\frac{1} {ca+2b^2+2b}\geq \frac{1}{ab+bc+ca} $

Tạm thời thế đã. Mọi người cùng giải nhé.

Conan Edogawa 01-09-2009 06:18 PM

Trích:

Nguyên văn bởi Red Devils (Post 43088)
Bài toán 2: (IMO Shortlist 2001)
Chứng minh rắng với mọi số thực dương $a, b, c $ ta có:
$\frac{a}{\sqrt{a^{2}+8bc}}+\frac{b}{\sqrt{b^{2}+8c a}}+\frac{c}{\sqrt{c^{2}+8ab}}\geq 1 $

Chuẩn hóa $abc=1 $. Khi đó BĐT cần cm tương đương:$\frac{1}{\sqrt{1+8a}}+\frac{1}{\sqrt{1+8b}}+\frac{ 1}{\sqrt{1+8c}}\ge 1 $

$\Leftrightarrow \sum{\sqrt{(1+8a)(1+8b)}}\ge \sqrt{(1+8a)(1+8b)(1+8c)} $

$\Leftrightarrow 8(a+b+c)+2\sqrt{(1+8a)(1+8b)(1+8c)}\sum{\sqrt{1+8a }\ge 510} $

Do $abc=1\Rightarrow a+b+c\ge 3 $ và $(1+8a)(1+8b)(1+8c)\ge 9{{a}^{\frac{8}{9}}}{{b}^{\frac{8}{9}}}{{c}^{\frac {8}{9}}}=729 $

Và $\sum{\sqrt{1+8a}\ge \sum{\sqrt{9{{a}^{\frac{8}{9}}}}\ge 9{{\left( abc \right)}^{\frac{4}{27}}}}=9} $

Cộng tất cả lại ta được đpcm.

trungdeptrai 01-09-2009 07:38 PM

Mở rộng Bài toán 2: (IMO Shortlist 2001)
Với $a,b,c\geq 0 $,và $k\geq 0 $,ta có:
$\frac{a}{\sqrt{{a}^{2}+kbc}}+\frac{b}{\sqrt{{b}^{2 }+kca}}+\frac{c}{\sqrt{{c}^{2}+kab}}\geq \frac{3}{\sqrt{1+k}} $.Mong mọi người thử...=p~=p~=p~

caube94 01-09-2009 09:06 PM

Trích:

Nguyên văn bởi Conan Edogawa (Post 43089)
Chuẩn hóa $abc=1 $. Khi đó BĐT cần cm tương đương:$\frac{1}{\sqrt{1+8a}}+\frac{1}{\sqrt{1+8b}}+\frac{ 1}{\sqrt{1+8c}}\ge 1 $

$\Leftrightarrow \sum{\sqrt{(1+8a)(1+8b)}}\ge \sqrt{(1+8a)(1+8b)(1+8c)} $

$\Leftrightarrow 8(a+b+c)+2\sqrt{(1+8a)(1+8b)(1+8c)}\sum{\sqrt{1+8a }\ge 510} $

Do $abc=1\Rightarrow a+b+c\ge 3 $ và $(1+8a)(1+8b)(1+8c)\ge 9{{a}^{\frac{8}{9}}}{{b}^{\frac{8}{9}}}{{c}^{\frac {8}{9}}}=729 $

Và $\sum{\sqrt{1+8a}\ge \sum{\sqrt{9{{a}^{\frac{8}{9}}}}\ge 9{{\left( abc \right)}^{\frac{4}{27}}}}=9} $

Cộng tất cả lại ta được đpcm.

Bài 2: Dùng holder cho gọn
Bài 3:
$a^8+b^8+c^8 \ge a^2b^2c^2(ab+bc+ca) $

Red Devils 01-09-2009 09:12 PM

Trích:

Nguyên văn bởi Conan Edogawa (Post 43089)
Chuẩn hóa $abc=1 $. Khi đó BĐT cần cm tương đương:$\frac{1}{\sqrt{1+8a}}+\frac{1}{\sqrt{1+8b}}+\frac{ 1}{\sqrt{1+8c}}\ge 1 $

Bạn làm rõ khúc này đi:)
------------------------------
Trích:

Nguyên văn bởi trungdeptrai (Post 43091)
Mở rộng Bài toán 2: (IMO Shortlist 2001)
Với $a,b,c\geq 0 $,và $k\geq 8 $,ta có:
$\frac{a}{\sqrt{{a}^{2}+kbc}}+\frac{b}{\sqrt{{b}^{2 }+kca}}+\frac{c}{\sqrt{{c}^{2}+kab}}\geq \frac{3}{\sqrt{1+k}} $

Nếu không nhầm thì phải là $k\geq 8 $
Lời giải:
Áp dụng BĐT Cauchy- Schwarz:
$\left(\sum \frac{a}{\sqrt{{a}^{2}+kbc}} \right)\left(\sum a\sqrt{a^2+kbc}\right)\geq \left(\sum a \right)^2 $
Tiếp tục áp dụng BĐT Cauchy- Schwarz:
${\left( {\sum {a\sqrt {{a^2} + kbc} } } \right)^2} = {\left( {\sum {\sqrt a \sqrt {{a^3} + kabc} } } \right)^2} \le \left( {\sum a } \right)\left( {\sum {({a^3} + kabc)} } \right) $
Do đó:
${\left( {\sum {\frac{a}{{\sqrt {{a^2} + kbc} }}} } \right)^2}\left( {\sum a } \right)\left( {\sum {({a^3} + kabc)} } \right) \ge {\left( {\sum {\frac{a}{{\sqrt {{a^2} + kbc} }}} } \right)^2}{\left( {\sum {a\sqrt {{a^2} + kbc} } } \right)^2} \ge {\left( {\sum a } \right)^4} $
$\Leftrightarrow {\left( {\sum {\frac{a}{{\sqrt {{a^2} + kbc} }}} } \right)^2} \ge \frac{{{{\left( {\sum a } \right)}^3}}}{{\left( {\sum {({a^3} + kabc)} } \right)}} $
Cần chứng minh:
$(k + 1){\left( {\sum a } \right)^3} \ge 9\left( {\sum {({a^3} + kabc)} } \right) $
$\Leftrightarrow \left( {k - 8} \right)\left( {{a^3} + {b^3} + {c^3}} \right) + 3\left( {k + 1} \right)\left( {a + b} \right)\left( {b + c} \right)\left( {c + a} \right) \ge 27kabc $
(đúng vì $k\geq 8 $)

Các bạn tích cực tham gia nhé, mình dự định cứ sau 20 bài sẽ tổng hợp thành 1 ebook nhỏ nhỏ, tất nhiên sẽ ghi tên các bạn tham gia post bài.:)

Hung_DHSP 01-09-2009 09:56 PM

Kazakhstan Intl 2006
 
Bài toán 5: Cho $a,b,c,d $ là các số thực có tổng bẳng $0 $.
CMR:
$ (ab+ac+ad+bc+bd+cd)^{2}+12\geq 6(abc+abd+acd+bcd). $

caube94 01-09-2009 10:06 PM

Cho $a;b;c \ge 0;a^2+b^2+c^2+abc=4 $.CMR:
$0 \le ab+bc+ca-abc \le 2 $
$\color{red}{USAMO} $ 2000

Red Devils 01-09-2009 10:10 PM

Trích:

Nguyên văn bởi Hung_DHSP (Post 43099)
Bài toán 5:(Kazakhstan Intl 2006)Cho $a,b,c,d $ là các số thực có tổng bẳng $0 $.
CMR:
$ (ab+ac+ad+bc+bd+cd)^{2}+12\geq 6(abc+abd+acd+bcd). $

Trích:

Nguyên văn bởi caube94 (Post 43104)
[U]Bài toán 6:Cho $a;b;c \ge 0;a^2+b^2+c^2+abc=4 $.CMR:
$0 \le ab+bc+ca-abc \le 2 $(\color{red}{USAMO}2000)

Ghi thứ tự bài và nguồn rõ ràng đi các bạn:)

trungdeptrai 01-09-2009 10:33 PM

bài 6:mình cũng không nhớX_X,nhưng ý tưởng sau hình như là của anh Cẩn.mình post cho mọi người tham khảo nha.
Trong ba số a-1,b-1,c-1 ;luôn có 2 ít nhất 2 số cùng dấu.Giả sử hai số đó là a-1,b-1.Ta có:$c\left(a-1 \right)\left(b-1 \right)\geq 0 $
$\Rightarrow abc\geq ac+bc-c $
Lại có:${a}^{2}+{b}^{2}\geq 2ab\Rightarrow {a}^{2}+{b}^{2}+{c}^{2}+abc\geq 2ab+{c}^{2}+abc $
$\Rightarrow ab\leq 2-c $
vậy$ab+bc+ca-abc\leq 2-c+bc+ca-\left(ac+bc-c \right)=2 $
Lại có:$ab+bc+ca-abc=ab\left(1-c \right)c\left(a+b \right)\geq 0 $
có dpcm...

Hung_DHSP 01-09-2009 10:57 PM

Trích:

Nguyên văn bởi caube94 (Post 43104)
Cho $a;b;c \ge 0;a^2+b^2+c^2+abc=4 $.CMR:
$0 \le ab+bc+ca-abc \le 2 $
$\color{red}{USAMO} $ 2000

Đây là USA MO 2001
Cũng là Bài toán 2.40 Sáng tạo bất đẳng thức.

Conan Edogawa 01-09-2009 11:14 PM

Người VN post bài VN chứ nhỉ=p~
Bài 7: (Olympic 30/4 năm 2006)
Cho a,b,c>0. Chứng minh:

$\frac{{{a}^{4}}}{{{a}^{4}}+\sqrt[3]{({{a}^{6}}+{{b}^{6}}){{({{a}^{3}}+{{c}^{3}})}^{2} }}}+\frac{{{b}^{4}}}{{{b}^{4}}+\sqrt[3]{({{b}^{6}}+{{c}^{6}}){{({{b}^{3}}+{{a}^{3}})}^{2} }}}+\frac{{{c}^{4}}}{{{c}^{4}}+\sqrt[3]{({{c}^{6}}+{{a}^{6}}){{({{c}^{3}}+{{b}^{3}})}^{2} }}}\le 1 $

trungdeptrai 02-09-2009 01:11 AM

Bài 8(IMO Shortlist 2000)a,b,c dương thỏa mãn abc=1.CM:
$\left(a-1+\frac{1}{b} \right)\left(b-1+\frac{1}{c} \right)\left(c-1+\frac{1}{a} \right)\leq 1 $
...:matrix:...

ll931110 02-09-2009 09:02 AM

Trích:

Nguyên văn bởi Hung_DHSP (Post 43099)
Bài toán 5: Cho $a,b,c,d $ là các số thực có tổng bẳng $0 $.
CMR:
$ (ab+ac+ad+bc+bd+cd)^{2}+12\geq 6(abc+abd+acd+bcd). $

Lâu không làm BĐT nhưng cũng thử chút xem sao :D
Xét phương trình
$f(x) = (x - a)(x - b)(x - c)(x - d) = 0 $
$ \leftrightarrow x^4 + (\sum ab)x^2 - (\sum abc)x + abcd = 0 $ (do $\sum a = 0 $)
$\rightarrow f'(x) = 4x^3 + 2(\sum ab)x - (\sum abc) = 0 (1) $

Do $f(x) $ có 4 nghiệm, nên theo định lí Rolle phương trình $f'(x) = 0 $ có 3 nghiệm, giả sử các nghiệm đó là $p,q,r $
Ta có:
$f'(x) = 4(x - p)(x - q)(x - r) = 4x^3 - 4(\sum p)x^2 + 4(\sum pq)x - 4pqr = 0 (2) $

Đồng nhất (1) và (2) ta được
$p + q + r = 0, \sum ab = 2 \sum pq, \sum abc = 4pqr $

Và BĐT đã cho tương đương với
$(\sum pq)^2 + 3 \ge 6pqr $
$\leftrightarrow \sum (pq)^2 + 3 \ge 6pqr $
$f(p,q,r) = \leftrightarrow \sum (pq)^2 + 3 - 6pqr \ge 0 $

BĐT hiển nhiên đúng nếu tồn tại 1 số = 0. BĐT cũng đúng nếu trong 3 số p,q,r có 1 số âm. Do vậy ta chỉ cần xét TH còn lại: có đúng 1 số dương trong 3 số p,q,r và không mất tính tổng quát, giả sử $p > 0 $

Tính $f(p,q,r) - f(p,\frac{q + r}{2}, \frac{q + r}{2}) = (q - r)^2.(\frac{7p^2}{16} - \frac{qr}{4} + \frac{3p}{4}) \ge 0 $
Cho nên BĐT sẽ đúng nếu ta cm được
$f(p,\frac{q + r}{2}, \frac{q + r}{2}) \ge 0 $
$\leftrightarrow 3p^4 - 8p^3 + 16 = 0 $
$\leftrightarrow (p - 2)^2(3p^2 - 4p + 4) \ge 0 $ (đúng)

Vậy ta có dpcm. Dấu đẳng thức xảy ra khi $p = 2, q = r = -1 $./.

Red Devils 02-09-2009 09:33 AM

Trích:

Nguyên văn bởi Conan Edogawa (Post 43109)
Bài toán 7: (Olympic 30/4 năm 2006)
Cho $a,b,c>0 $. Chứng minh:

$\frac{{{a}^{4}}}{{{a}^{4}}+\sqrt[3]{({{a}^{6}}+{{b}^{6}}){{({{a}^{3}}+{{c}^{3}})}^{2} }}}+\frac{{{b}^{4}}}{{{b}^{4}}+\sqrt[3]{({{b}^{6}}+{{c}^{6}}){{({{b}^{3}}+{{a}^{3}})}^{2} }}}+\frac{{{c}^{4}}}{{{c}^{4}}+\sqrt[3]{({{c}^{6}}+{{a}^{6}}){{({{c}^{3}}+{{b}^{3}})}^{2} }}}\le 1 $

Lời giải: Áp dụng BĐT Holder ta có:
$\sqrt[3]{(a^6+b^6)(a^3+c^3)^2}=\sqrt[3]{(a^6+b^6)(c^3+a^3)(c^3+a^3)}\geq \left(\sqrt[3]{a^6.c^3.c^3} \right)+\left(\sqrt[3]{b^6.a^3.a^3} \right)=a^2c^2+a^2b^2 $
Tương tự:$ \sqrt[3]{({{b}^{6}}+{{c}^{6}}){{({{b}^{3}}+{{a}^{3}})}^{2} }}\geq a^2b^2+b^2c^2 $
$\sqrt[3]{({{c}^{6}}+{{a}^{6}}){{({{c}^{3}}+{{b}^{3}})}^{2} }}\geq b^2c^2+c^2a^2 $
Suy ra: $\frac{{{a}^{4}}}{{{a}^{4}}+\sqrt[3]{({{a}^{6}}+{{b}^{6}}){{({{a}^{3}}+{{c}^{3}})}^{2} }}}+\frac{{{b}^{4}}}{{{b}^{4}}+\sqrt[3]{({{b}^{6}}+{{c}^{6}}){{({{b}^{3}}+{{a}^{3}})}^{2} }}}+\frac{{{c}^{4}}}{{{c}^{4}}+\sqrt[3]{({{c}^{6}}+{{a}^{6}}){{({{c}^{3}}+{{b}^{3}})}^{2} }}}\leq \sum \frac{{{a}^{4}}}{{{a}^{4}}+a^2c^2+a^2b^2}=\sum \frac{a^2}{a^2+b^2+c^2}=1 $

Trích:

Nguyên văn bởi trungdeptrai (Post 43111)
Bài toán 8: (IMO Shortlist 2000) Cho $a,b,c $ dương thỏa mãn $abc=1 $.CM:
$\left(a-1+\frac{1}{b} \right)\left(b-1+\frac{1}{c} \right)\left(c-1+\frac{1}{a} \right)\leq 1 $

Lời giải: Vì $abc=1 $ nên đặt $a=\frac{y}{z},b\frac{z}{x},c=\frac{x}{y} $. Bất đẳng thức cần chứng minh trở thành: $(-x+y+z)(x-y+z)(z+y-z)\leq xyz $ (Đây là BĐT Schur)

trungdeptrai 02-09-2009 10:08 AM

Bài 9(MOP 02)Với a,b,c dương.CM:
${\left(\frac{2a}{b+c} \right)}^{\frac{2}{3}}+{\left(\frac{2b}{c+a} \right)}^{\frac{2}{3}}+{\left(\frac{2c}{a+b} \right)}^{\frac{2}{3}}\geq 3 $


Múi giờ GMT. Hiện tại là 11:44 AM.

Powered by: vBulletin Copyright ©2000-2024, Jelsoft Enterprises Ltd.

[page compression: 24.83 k/26.47 k (6.20%)]